Pediatric EOR

Ace your homework & exams now with Quizwiz!

BIsotonic saline intravenous infusion presenting with moderate to severe hypovolemia and needing a more urgent tx

A 4-year-old girl presents to the clinic after 3 days of persistent vomiting and intermittent diarrhea. Her mother states she has had a temperature of 100.7°F for the past 24 hours and describes her child as irritable and lethargic. The patient has no known underlying medical conditions. Physical examination reveals HR of 142 bpm, RR of 28 breaths per minute, T of 100.9°F, and oxygen saturation of 98% on room air. Skin is cool, capillary refill is delayed, and the patient has decreased skin turgor. Which of the following is appropriate treatment at this time? AIntravenous ceftriaxone BIsotonic saline intravenous infusion COndansetron administration DOral rehydration with an oral electrolyte solution

AHydrogen breath test Although an upper gastrointestinal endoscopy with small bowel biopsies is considered the gold standard to diagnose lactose intolerance, it is rarely used because of the invasive nature and high cost of the test.

1st line test to detect lactose deficiency? AHydrogen breath test BLactose tolerance test CSmall bowel biopsy DUrea breath test

BIndomethacin Patent ductus arteriosus (PDA) is an acyanotic heart disease that is characterized by the failure of normal closure of the ductus arteriosus after birth. It is kept patent by low arterial oxygen and circulating prostaglandin E2 (which is mediated by cyclooxygenase-2).

2 day old pre-term grade 3 continuous machine-like murmur heard in the left infraclavicular area and bounding radial and brachial pulses bilaterally. with poor feeding and respiratory distress. what is most appropriate tx AFurosemide BIndomethacin CObservation DSurgical ligation

CTarget sign and increased pyloric muscle thickness Pyloric stenosis-- young infants caused by hypertrophy of the pylorus, which can lead to gastric outlet obstruction and vomiting. Risk factors for pyloric stenosis include male sex, preterm birth, maternal smoking. The classic presentation consists of a 3- to 6-week-old infant with postprandial, projectile, and nonbilious vomiting, hungry after vomiting. palpable olive-like mass in right upper quadrant. DX: Abdominal ultrasound is the preferred diagnostic test to confirm pyloric stenosis. The typical findings are a target sign on a transverse view and increased pyloric muscle thickness, pyloric muscle length, and pyloric diameter. TX: surgically with pyloromyotomy

4 week old preterm boy w postprandial, projectile, and non bilious vomiting. Small firm round mass in RUQ. Which of the following is the expected finding? AGallstones BLayers of the intestine within the intestine CTarget sign and increased pyloric muscle thickness DWhirlpool sign of mesentery

ABrown rings around the iris Kayser-Fleischer rings (brownish rings that encircle the iris) are a classic finding visible in some patients with hepatic Wilson disease and almost all patients with neurologic Wilson disease. copper accumulates in liver kidneys brain and eyes.

A 15-year-old boy presents to the clinic for follow-up on elevated liver enzymes found incidentally on laboratory testing. His aspartate aminotransferase is 112 IU/L, and his alanine aminotransferase is 78 IU/L. Which of the following findings would most support the diagnosis of Wilson disease? ABrown rings around the iris BPalpable organ in the left upper quadrant of the abdomen CPositive fluid wave test DYellow discoloration of the skin

BHerpetic gingivostomatitis

A 3-year-old girl presents to the clinic with diffuse ulcerative lesions on her oral mucosa and around her lips. Her mother reports she has had a fever, decreased appetite, malaise, and irritability for the past 2 days. She has no prior medical history and has not started any new medications recently. Physical exam reveals yellow ulcerative lesions with a red halo on her buccal mucosa, lips, and perioral area. There are no other cutaneous lesions. Which of the following is the most likely diagnosis? AAphthous ulcers BHerpetic gingivostomatitis COral candidiasis DStevens-Johnson syndrome

ACentral erythematous area surrounded by pale edematous skin and a sharp discrete ring of erythema Erythema multiforme is an inflammatory dermatologic disease characterized by target-shaped skin lesions. It is most commonly associated with herpes simplex, Mycoplasma pneumoniae, and upper respiratory tract infections. Other causes include contact allergens, medications (e.g., antiepileptics, nonsteroidal anti-inflammatory drugs, sulfonamides, antibiotics), connective tissue diseases, X-ray therapy, pregnancy, and internal malignancies. target lesions, erythematous papules and macules, urticaria-like lesions, vesicles, and bullae. Target lesions initially present as dusky red, round macules and papules with pruritus and burning that evolve to a central erythematous area surrounded by a pale area of edematous skin and a sharp discrete ring of erythema past this pale area.

A 12-year-old boy presents to the clinic for evaluation of a new-onset pruritic rash that appeared in symmetric crops on his palms, soles, and the extensor aspect of his forearms. He recently recovered from a viral upper respiratory infection, for which he was treated conservatively using nonsteroidal anti-inflammatory drugs. The patient reports no current systemic symptoms, but he does have some oral discomfort with associated bullae noted in the oral cavity. Which of the following most likely describes the patient's skin findings? ACentral erythematous area surrounded by pale edematous skin and a sharp discrete ring of erythema BLarge, annular erythematous lesion with a necrotic center found on the popliteal space COval erythematous, scaling plaque with a central clearing and an active raised border DRound salmon-colored lesion with central clearing and a collarette of scale

BAtopic dermatitis Nasal polyposis, asthma, and aspirin hypersensitivity complete a constellation of conditions known as the Samter triad that represents aspirin-exacerbated respiratory disease (AERD).

A 12-year-old boy presents to the clinic with a 6-month history of intermittent wheezing, cough, and shortness of breath. He states he notices the symptoms are worse after recent exacerbation of his allergy symptoms. Physical exam reveals nasal polyposis. Which of the following conditions completes the classic triad of diseases associated with the suspected diagnosis? AAspirin hypersensitivityCorrect Answer BAtopic dermatitis CEosinophilic esophagitis DVasomotor rhinitis

DPansystolic murmur Rheumatic fever is an infectious disease causing systemic inflammation approximately 2 to 3 weeks after an episode of pharyngitis. Caused by group A Streptococcus, it is prevalent in transitional areas of the world and is the leading cause of cardiovascular death during the first 50 years of life. Rheumatic fever most frequently affects children between ages 5 and 15 years old. Mitral regurgitation is the most common manifestation of this complication, which presents on cardiac auscultation as a pansystolic murmur.

A 13-year-old girl presents to the clinic to follow up after being diagnosed with acute pharyngitis 1 month ago. She reports new-onset joint aches that have migrated from her bilateral knees to her hips over the past week. Additionally, she reports nodules have formed over her olecranon process in her right elbow. Physical examination reveals fever, subcutaneous nodules, and mild effusion in the bilateral knees. Which of the following is the most common valvular finding during cardiac auscultation and is associated with pathologic sequelae? AEarly diastolic high-frequency snap BHarsh, systolic crescendo-decrescendo murmur CMidsystolic click DPansystolic murmur

DSupportive treatment at home with precautions Bronchiolitis is a lower respiratory tract disease and a major cause of illness and hospitalization in children under 2 years of age that most often results from infection with a viral pathogen. Typically, bronchiolitis begins with upper respiratory manifestations, such as fever, nasal congestion, and rhinorrhea, then progresses to lower respiratory infection and inflammation, resulting in wheezing and rales with or without respiratory distress. Severe bronchiolitis is generally indicated by any of the following: persistently increased respiratory effort (tachypnea, nasal flaring, intercostal, subcostal, or suprasternal retractions, accessory muscle use, grunting), hypoxemia (SpO2 < 95%), apnea, and acute respiratory failure.

A 14-month-old boy presents to the clinic with his mother, who reports a 3-day history of progressive cough, runny nose, and wheezing. Exam reveals an afebrile patient with respirations of 30/minute and oxygen saturation of 96% on room air. Exam also reveals moderate expiratory wheeze in all lung fields and mild clear rhinorrhea in bilateral nares. Which of the following is the most appropriate treatment plan for the suspected diagnosis? AAdmission to the nearest hospital BHigh-flow nasal cannula and monitoring CInhaled bronchodilator and monitoring DSupportive treatment at home with precautions

DMetabolic alkalosis with hypokalemia

A 14-year-old girl presents to the clinic with lethargy, irregular menses, and abdominal pain with bloating. Physical examination reveals sialadenitis and Russel sign. She reports no use of medications or supplements. Which of the following is most likely found on laboratory studies? AMetabolic acidosis with hyperkalemia BMetabolic acidosis with hypokalemia CMetabolic alkalosis with hyperkalemia DMetabolic alkalosis with hypokalemia

DHypophosphatemia Nutritional rehabilitation is an essential component of initial treatment, however, an overly aggressive approach in a malnourished patient can lead to refeeding syndrome. The hallmark of this condition is hypophosphatemia.

A 15-year-old girl presents to the clinic with amenorrhea and easy fatigability. She was 13 years old at menarche and states she had normal menstrual cycles until 3 months ago. Vitals today reveal HR 48 bpm, RR 19 breaths/min, BP 99/62 mm Hg, T 94.9°F, and SpO2 99% on room air. Her calculated body mass index is 16.2 kg/m2, and physical examination demonstrates an emaciated appearance with fine, downy body hair. You decide to implement outpatient nutritional rehabilitation. Which of the following is the hallmark laboratory finding indicating overly aggressive initial outpatient therapy? AHypokalemia BHypomagnesemia CHyponatremia DHypophosphatemia

AStart acetaminophen as needed The patient in the vignette has infectious mononucleosis, also known as Epstein-Barr virus (EBV). This is a viral illness that is most commonly shed via salivary secretions and thus is sometimes referred to as the kissing disease. Infectious mononucleosis commonly presents with a triad of fever, tonsillar pharyngitis, and posterior cervical lymphadenopathy. if patients with infectious mononucleosis are treated with amoxicillin or ampicillin, they may develop a maculopapular, urticarial, or petechial rash. This rash is not a true allergy to the antibiotic, rather it is a transient hypersensitivity reaction to the antibiotic previously given. Diagnosis is made clinically, but the diagnosis should be confirmed via heterophile antibody testing or by EBV-specific antibodies.

A 15-year-old girl with a known allergy to ibuprofen presents to the emergency department with concerns of a new rash. She was evaluated by her primary care physician 8 days ago after developing a fever and pharyngitis. She was started on amoxicillin for suspected strep throat. Per the patient, she first noticed the rash this morning. On physical exam, the patient has posterior cervical lymph nodes, and her tonsils are 3 in size, with white exudate and surrounding erythema. She also has a generalized maculopapular rash. Which of the following is the most appropriate treatment for this patient, after discontinuing amoxicillin? AStart acetaminophen as needed BStart dexamethasone every 6 hours for 7 days CStart nonsteroidal anti-inflammatory drugs as needed DSwitch to amoxicillin-clavulanate twice a day for 7 days

CTopical corticosteroid therapy Pityriasis rosea is a benign, self-limiting rash that occurs in otherwise healthy individuals and is common in children and young adults.

A 16-year-old girl presents to the office for a moderately pruritic and bothersome rash that appeared on her abdomen and back 2 days ago. On physical examination, she has a diffuse rash that is characterized by hypopigmented macules and papules that are bilateral and symmetrical, following the lines of cleavage of the skin. The patches appear fine and scaly but are not tender to palpation. There is also a round, salmon-colored patch on her back that is well demarcated that she has never noticed before. Which of the following is the best initial treatment, besides reassurance? AOral antiviral therapy BTopical antifungal therapy CTopical corticosteroid therapy DUltraviolet radiation therapy

CReed-Sternberg cells Hodgkin lymphoma is a malignant lymphoma that most commonly affects young adults, in particular those between 15 to 19 years of age. Hodgkin lymphoma is diagnosed through tissue biopsy, which demonstrates the pathognomonic Reed-Sternberg cells.

A 16-year-old girl presents with fever, pharyngitis, generalized lymphadenopathy, and sudden-onset fatigue. She is treated appropriately and, although she has had resolution of her primary symptoms, the patient now reports dyspnea and dysphagia. Physical exam demonstrates persistent, painless cervical lymphadenopathy, while chest radiography reveals a mediastinal mass. Which of the following diagnostic findings is pathognomonic for her suspected condition? AHypercellular bone marrow aspirate BPhiladelphia chromosome CReed-Sternberg cells DSmudge cells on peripheral blood smear

BSerum bicarbonate of 13 mEq/L Children with DKA typically have blood glucose levels over 200 mg/dL, a marked metabolic acidosis with venous pH < 7.3 or a serum bicarbonate < 15 mEq/L, and the presence of ketones in the blood.

A 4-year-old boy presents to the clinic with his parents. He has been concerned by vague abdominal pain, nausea, vomiting, and anorexia for the last several days. He is urinating more frequently and notes excessive thirst despite drinking lots of clear fluids. He has been irritable and lethargic. On physical examination, his mucous membranes appear dry, and he appears fatigued. A blood glucose level is 580 mg/dL. Which of the following lab abnormalities would be consistent with the most likely diagnosis? AAnion gap of 14 mmol/L BSerum bicarbonate of 13 mEq/L CSerum sodium of 147 mEq/L

DTrisomy 21 Hirschsprung disease is a congenital defect of the colon due to the failure of ganglia to develop in the distal colon. It is characterized by the lack of peristalsis and involuntary relaxation of the internal anal sphincter, leading to bowel obstruction. commonly found in boys associated with trisomy 21. Failure to pass meconium in the first 24 hours of life. . A physical exam will reveal abdominal distension and a very tight anal sphincter upon digital rectal examination. An abdominal X-ray will show dilated loops of bowel without gas or stool present in the rectum. A contrast enema may reveal a narrowed distal colon with proximal dilation. DX: biopsy

A 2-day-old infant boy is evaluated for poor feeding and abdominal distention. He has had several episodes of bilious vomiting and has not passed any meconium since birth. An abdominal radiograph reveals dilated loops of bowel without gas or stool present in the rectum. Which of the following comorbidities is associated with this patient's condition? ABeckwith-Wiedemann BMitral valve prolapse CSpina bifida DTrisomy 21

BLow birth weight Acute bronchiolitis is a contagious infection of the lower respiratory system involving the bronchioles. It is most frequently caused by respiratory syncytial virus (RSV) and less commonly by rhinovirus, parainfluenza virus, adenovirus, influenza virus, and coronavirus. Risk factors for developing this condition include age < 3 months, low birth weight, gestational age (especially infants born prior to 29 weeks of gestation), low socioeconomic status, childcare attendance, tobacco smoke exposure, cesarean delivery, chronic lung disease or airway insufficiency, and congenital heart, lung, or immune deficiency.

A 2-month-old girl presents with a runny nose, persistent wet-sounding cough, and wheezing for the past 5 days that has not improved. She is having a difficult time feeding from a bottle, seems to be gasping for air between latches, and cries out of frustration. Upon physical exam, she has a fever of 100.7°F, mild retractions, and diffuse fine rales in all lung fields revealed on auscultation. Her chest radiograph is unremarkable, and a nasal secretion sample is taken to be tested. Which of the following is a risk factor for the suspected condition? AFormula feeding instead of breastfeeding BLow birth weight CPost-term birth DVaginal delivery

AAtrial septal defect Therefore, a midsystolic murmur is often best heard at the second left intercostal space in patients with moderate or large atrial septal defects. Another common cardiac exam finding is a fixed and split S2 sound due to the aortic valve closing prior to the pulmonic valve. An echocardiogram is the diagnostic test of choice. Most atrial septal defects resolve spontaneously during infancy or early childhood. However, surgical patching is indicated in patients with clinically significant symptoms, such as failure to thrive.

A 2-month-old girl presents with her mother to the clinic for a routine childhood exam. You auscultate her heart and note a midsystolic murmur best heard at the second left intercostal space and a fixed and split S2. Which of the following is the most likely diagnosis? AAtrial septal defect BCoarctation of the aorta CPatent ductus arteriosus DVentricular septal defect

CObservation and massage Congenital nasolacrimal duct obstruction (dacryostenosis) is the blockage of the nasolacrimal duct that results in persistent tearing and ocular discharge in infants and young children. spontaneous resolution by 6 months of age, although a small percentage may persist after this period. If the disorder persists beyond 12 months of age, it is unlikely to resolve spontaneously. In most cases, the treatment of nasolacrimal duct obstruction consists of nonsurgical management, including observation and massage (Crigler massage).

A 2-month-old infant presents with his mother, who states the child has been experiencing persistent tearing and debris on the eyelashes since birth. On exam, you note reflux of tears onto the eyes when the medial aspect of the eye is palpated. A fluorescein dye disappearance test is positive. Which of the following is the most appropriate treatment for this patient given the most likely diagnosis? ALacrimal duct probing BNasolacrimal intubation CObservation and massage DTopical antibiotics

A8 (VIII) aka Christmas disease B9 (IX)

Hemophilia A and B deficiency factors?

BModerate Mild dehydration is defined by a 3-5% volume depletion, minimal symptoms, including slightly dry mucous membranes, increased thirst, and sometimes mild reduction in urine output. Moderate dehydration is defined by a 6-9% volume depletion and presents clinically with tachycardia, deep respiration, dry mucous membranes, sunken anterior fontanel (in infants), sunken eyes, reduced skin turgor, increased capillary refill time (2-3 seconds), cool skin, reduced urine output, and irritability. Severe dehydration is defined by at least 10% volume loss, manifests clinically as tachycardia, hypotension, deep tachypnea or decreased breathing, parched mucous membranes, markedly sunken anterior fontanel, markedly sunken eyes, tenting of skin turgor, cool and mottled skin, anuria, and lethargy. Tx: fluid resuscitation. Severe: 20 mL/kg of isotonic intravenous fluids. Mild/Moderate: oral fluids.

A 2-year-old boy presents to the clinic with 2 days of vomiting and diarrhea. Vital signs include a T of 98.6°F, HR of 155 bpm, RR of 24 breaths/minute, and a blood pressure of 100/60 mm Hg. Physical exam reveals dry mucous membranes, sunken eyes, delay in capillary refill to 2.5 seconds, and a soft and nontender abdomen. Which of the following describes the severity of dehydration in this patient? AMild BModerate CSevere DVery severe

DType IV hypersensitivity Most drug exanthems are caused by delayed-type T cell-mediated type IV immune reactions. This response is understood to activate macrophages, eosinophils, or neutrophils, and different drugs likely harness different mechanisms in their activation of the immune system.

A 2-year-old boy presents to the emergency department with a new-onset rash. His vital signs are within normal limits, and he is in no acute distress, though he does appear irritated. His parents report no known history of medication allergies, and there is no personal or family history of asthma, eczema, or environmental allergies. He attends daycare and has a prior history of ear infections, most recently completing a 7-day course of amoxicillin 2 days ago for a right ear infection. Otherwise, his medical history is unremarkable. On exam, he has a diffuse, morbilliform rash over his trunk and bilateral upper extremities. There are no pustules or vesicles. A few spots seem to be cropping up on his face, and his palms and soles are spared. There is no mucosal involvement, and there is no swelling or erythema over his joints. Other than the rash, he has a normal exam, is consolable, and appears well. Which of the following is the type of reaction causing this rash? AType I hypersensitivity BType II hypersensitivity CType III hypersensitivity DType IV hypersensitivity

CNeurons Herpes simplex virus (HSV) can be divided into two types: type 1 (HSV-1) and type 2 (HSV-2). HSV-1 is typically associated with disease to the oral area, and HSV-2 is usually associated with genital disease, although either virus can cause lesions in both locations. The virus can remain latent in the nerve cell ganglia

A 2-year-old boy presents to the office with a fever, low energy, and decreased appetite that began yesterday. On physical exam, he has marked erythema of the lips and oral mucosa with noticeable bleeding along the gum lines. Small vesicles are noted on the lower lip, along with tender cervical lymphadenopathy. He is given a prescription for acyclovir, and his caretakers are told that the virus causing this infection will enter a latent stage but can reactivate in the future. At which one of the following sites does this virus establish latent infection? AB cells BMyeloid progenitor cells CNeurons DT cells

DVoiding cystourethrography (VCUG). Vesicoureteral reflux is best diagnosed by diagnostic imaging tools such as kidney and bladder ultrasonography and voiding cystourethrography (VCUG). VCUG is the criterion standard for diagnosis. Vesicoureteral Reflux Hydronephrosis on U/S Primary: incompetent UVJ closureNon-toilet-trained children with febrile UTI, prenatal Dx

A 2-year-old girl with a history of two febrile urinary tract infections over the past 6 months presents to the office for workup for vesicoureteral reflux. Her mother states that the patient's older brother was also diagnosed with the condition as a child and it resolved spontaneously. Which of the following is the study of choice for the diagnosis of this condition? ABladder ultrasonography BCystoscopy CUrodynamic study DVoiding cystourethrography

AErythema toxicum neonatorum Erythema toxicum neonatorum is a dermatologic disorder commonly seen in neonates within the first 72 hours after birth but may develop anytime between 1 and 5 days after birth. Risk factors include higher birth weight and greater gestational age. The etiology is unknown, present with multiple erythematous macules and papules on the trunk, face, and proximal extremities that spare the palms and soles. These macules and papules typically appear within 24-48 hours of birth, progress to pustules on an erythematous base, and eventually resolve within 1 week. A presumptive clinical diagnosis is usually made based on the timing and appearance of the pustules. Definitive diagnosis can be made with a Wright-stained smear. No treatment is required

A 3-day-old baby girl born at 41 weeks gestation via a normal spontaneous vaginal delivery is evaluated in the labor and delivery unit for a new rash. Her mother states she noticed the small bumps for the first time today when changing her diaper. Physical examination reveals multiple erythematous macules and papules on the cheek, anterior trunk, upper arm, and thighs. The rash on the cheek is shown above. No lesions are noted anywhere else on the body. Which of the following is the most likely diagnosis, given the timing, location, and appearance of the infant's lesions? AErythema toxicum neonatorum BMilia CMiliaria crystallina DNeonatal cephalic pustulosis

CFecal-oral Congenital toxoplasmosis is caused by the protozoan parasite Toxoplasma gondii. T. gondii oocysts in the soil or tissues of small prey are ingested by cats and replicate in the feline intestines. These oocysts are excreted in the feces and are ingested by humans through the fecal-oral route. The classic triad of congenital toxoplasmosis is chorioretinitis, intracranial calcifications, and hydrocephalus.

A 3-day-old boy born to a mother with no prenatal care is being evaluated for fever, anemia, and thrombocytopenia. Ophthalmologic evaluation is significant for chorioretinitis, and a head computed tomography scan reveals intracranial calcifications and hydrocephalus. Placental transmission from the mother to the newborn is highly suspected. Which of the following best characterizes the route of transmission associated with maternal infection of the most likely organism? AAerosol BDirect contact CFecal-oral DFomite

CLoss of ganglion cells in the distal colon Hirschsprung disease (also called congenital aganglionic megacolon) is a motility disorder of the gut due to the failure of neural crest cells (precursor of enteric ganglion cells) to migrate completely during fetal intestinal development, resulting in functional obstruction. Neonates present with meconium ileus (failure to pass meconium in the first 48 hours of life) accompanied by bilious vomiting and abdominal distention. s/sx: enterocolitis, including fever, nausea, vomiting, and abdominal distention, which can progress to sepsis and megacolon. PE: squirt sign (explosive expulsion of gas and stool after digital rectal exam). Xray: dilated loops of bowel proximal to the aganglionic segment and decreased or absent air in the rectum. def dx: rectal biopsy, absence of ganglion cells in the rectosigmoid colon. tx: surgical resection of the affected bowel.

A 3-day-old neonate with Down syndrome presents with bilious vomiting, abdominal distention, and low-grade fever. He has not passed meconium since birth. On exam, you note abdominal distention. Digital rectal exam results in an explosive expulsion of stool. An abdominal radiograph reveals dilated loops of bowel and absence of air in the rectum, whereas contrast enema shows transition zone and caliber change between the normal and affected bowel. Which of the following is the most likely cause of the suspected diagnosis? AComplete absence of a portion of the duodenum BHyperplasia and hypertrophy of the pylorus muscle CLoss of ganglion cells in the distal colon DTelescoping of the proximal bowel into the distal bowel

BNumerous eosinophils Erythema toxicum neonatorum is a benign pustular disorder seen in neonates, particularly in term neonates and neonates with higher birth weight. Skin eruption typically occurs in the first 24 to 48 hours of life, although delayed eruption as late as 10 days has been described. The diagnosis is made clinically by the presence of the characteristic features and confirmed by a microscopic exam of a wright-stained smear of the contents of a pustule that demonstrates numerous eosinophils and occasional neutrophils. No treatment is necessary, as the illness resolves spontaneously.

A 3-day-old term neonate presents with a rash his mother noticed 10 hours ago. On exam, you note blotchy erythematous patches with central pustules on the trunk and extremities without the involvement of the palms or soles. The rest of the physical exam is unremarkable. Which of the following findings would be seen on a microscopic examination of a wright-stained smear of the contents of a pustule? AMultinucleated giant cells BNumerous eosinophils CNumerous neutrophils DOccasional eosinophils

DSurgical pyloromyotomy A target sign is also seen on a transverse view of an abdominal ultrasound. Barium enema upper GI series may show a string sign, which is caused by barium moving through an elongated, thickened pylorus. The definitive treatment for pyloric stenosis requires surgical management (surgical pyloromyotomy). The surgery involves a longitudinal incision of the hypertrophied pylorus, with blunt dissection to the level of the submucosa, thereby relieving the constriction and allowing the passage of food into the duodenum.

A 3-week-old infant presents with his father, who states the infant has been experiencing forceful vomiting after each feed for 2 days. On exam, you note a well-hydrated and well-nourished infant. You also note a palpable olive-like mass on the lateral edge of the rectus abdominis muscle in the right upper quadrant. Laboratory studies are unremarkable. A target sign is seen on a transverse view of an abdominal ultrasound. Which of the following is the most appropriate clinical intervention? AAnticholinergic management BBalloon dilation CPneumatic reduction DSurgical pyloromyotomy

AHyperinflation of the left lung Foreign body aspiration is a serious, potentially fatal event that is most commonly found in children under 3 years of age. A physical exam may reveal unilateral wheezing, decreased breath sounds, stridor, and cyanosis. A chest radiograph may show unilateral hyperinflation, mediastinal shift, and atelectasis. There may be focused areas of pleural effusion, hilar adenopathy, bronchial thickening, and bronchiectasis near the site of the foreign body. Most foreign bodies will not be visualized.

A 3-year-old boy presents to the emergency department because he started gasping and coughing after he was eating popcorn about 20 minutes ago. Upon physical exam, the patient is mildly distressed but afebrile. He has a cough that is off and on, has mild stridor, and there are decreased breath sounds in the lower left posterior lung field. Which of the following would most likely be found on a chest radiograph? AHyperinflation of the left lung BLeft lobar pneumonia CMediastinal shift toward the left side DRadiopacity to the left lung field representing the foreign body

DRetropharyngeal abscess sore throat, dysphagia, and fever. However, as the disease progresses, the patient will also develop neck stiffness and stridor. The diagnosis is confirmed by a contrast computed tomography scan of the neck, which would note the presence of pus to the retropharyngeal space. Primary treatment includes securing the patient's airway, followed by a surgical incision and drainage along with empiric IV antibiotics. The most common antibiotics used are ampicillin-sulbactam or clindamycin, which would cover group A Streptococcus, Staphylococcus aureus, and other respiratory anaerobes.

A 3-year-old girl presents to the emergency department accompanied by her mother. She notes the patient has not been wanting to eat or drink and has been having intermittent fevers for the past week. She is most concerned because she has noticed the patient has not wanted to move her neck today. The mother notes the patient had a viral upper respiratory infection about 10 days ago that resolved on its own. She is up to date on her vaccinations. On physical exam, the patient has nuchal rigidity and mild inspiratory stridor. There is diffuse erythema and edema to the posterior oropharynx. Her voice is muffled, and she is crying and calling for her mother. An X-ray was taken that showed prevertebral soft tissue swelling on the lateral view. Which of the following is the most likely diagnosis? ACroup BEpiglottitis CPeritonsillar abscess DRetropharyngeal abscess

D Rooting reflex This reflex is present at birth in term newborns (38-40 gestational weeks) and typically disappears at 2-3 months of age.

A 4-month-old girl presents to the office for her well-child checkup. She has reached her developmental milestones on time and has no known medical conditions. Which of the following reflexes would be most likely to have disappeared by today's physical examination? AGalant reflex BParachute reflex CPlantar grasp DRooting reflex

CErythromycin Infantile hypertrophic pyloric stenosis is characterized by new-onset, nonbilious, forceful vomiting in a 3- to 6-week-old infant. Risk factors include genetic predisposition, neonatal hypergastrinemia, prematurity (< 37 weeks gestation), maternal smoking during pregnancy, bottle feeding, first-born children, and male sex. Associated triggers include macrolide antibiotics such as erythromycin or azithromycin administered to the mother or infant, infant exposure to cow milk or soy proteins, and infant exposure to caretakers with diarrheal disease. The infant is often described as a hungry vomiter, desiring to be refed soon after vomiting. Physical examination often reveals an emaciated, dehydrated infant. A palpable olive-like mass in the right upper quadrant is pathognomonic. DX: US (target sign**) TX: supportive

A 4-week-old boy born at 35 weeks gestation to a G1P1 mother presents to his pediatrician for nonbilious, projectile vomiting over the last week. His mother states he always seems hungry. Dry mucous membranes and decreased skin turgor are noted on physical examination. Which of the following medications taken during pregnancy is associated with the patient's condition? ACeftriaxone BClindamycin CErythromycin DTrimethoprim-sulfamethoxazole

BObserve closely and follow up Immune thrombocytopenia (ITP), formerly known as immune thrombocytopenic purpura, is a condition characterized by isolated thrombocytopenia that occurs in otherwise healthy individuals. he goal of treatment is to get the platelet count to a safe level. Most children have spontaneous resolution and do not require treatment, thus close observation and follow-up would be the most appropriate option for the child in the vignette. Children with moderate to severe symptoms can be given corticosteroids (e.g., prednisone, methylprednisolone, dexamethasone) or IV immune globulin (IVIG) with rituximab. Those with life-threatening active bleeding and a platelet count of < 30,000 platelets/µL should be immediately hospitalized and given IVIG followed by a platelet transfusion

A 4-year-old boy presents to the office with tiny dark red spots on his arms and legs for the past 3 days. The patient is afebrile, pleasant, and cooperative on physical examination. There are multiple round pinpoint macules that are nonblanching and nontender to palpation and are sparsely distributed on his bilateral upper and lower extremities. He had an episode of acute gastroenteritis about 1 week ago but is otherwise healthy. His uncle had similar symptoms around this age, as well. He has a platelet count of 95,000 platelets/µL with a hemoglobin of 12.1 g/dL. Which of the following is the best next step in treatment for this patient? AAdmit to the hospital to administer IV immune globulin therapy BObserve closely and follow up CStart oral prednisone DStart rituximab infusion

BRecommend an oral laxative Encopresis is the involuntary loss of stool typically found in children who have already been toilet trained. It is usually due to overflow incontinence as a result of chronic constipation, although some patients do not have a history of constipation. Patients will have a history of painful defecation and episodes of underwear soiling. Colonic disimpaction followed by long-term oral laxative therapy is recommended.

A 4-year-old boy who has been toilet trained for about 1 year has had multiple episodes of fecal soiling over the past several months. The episodes are unpredictable but mainly occur during the day. His caretaker reports that he occasionally has hard, painful stools and often does not want to try to defecate because of the pain. On physical exam, the patient is tearful but well-nourished, well-developed, and cooperative. A digital rectal exam reveals hardened stool palpable in the rectal vault and some loose, soft stool noted on his underwear. Which of the following is the best initial step in treatment? ARecommend a fiber supplement BRecommend an oral laxative CRecommend stricter toilet training habits DRefer him for biofeedback therapy

CIntravenous immune globulin with high-dose aspirin Kawasaki disease, also known as mucocutaneous lymph node syndrome. Kawasaki disease is a self-limiting vasculitis of the small and medium vessels with unknown etiology. It primarily affects boys younger than 5 years of age. CRASH and burn: Conjunctivitis, rash, adenopathy, strawberry tongue, hands and feet swelling + fever >/=5 days

A 4-year-old boy who is unimmunized presents to the clinic with a 5-day history of fever. Vital signs reveal HR of 110 bpm, BP of 100/60 mm Hg, and T of 102.5°F. Physical exam reveals bilateral conjunctivitis, erythema of the hands and feet, a diffuse morbilliform rash, and the finding shown above. Which of the following is the most appropriate initial treatment for the suspected condition? AAcetaminophen BDiphenhydramine CIntravenous immune globulin with high-dose aspirin DPenicillin V

B Excess skin at the nape of the neck Down syndrome is the most common chromosomal disorder (trisomy 21). upslanting palpebral fissures; prominent epicanthic folds; brachycephaly; excess skin at the nape of the neck; a flat facial profile; low-set, small ears; a protruding tongue; a narrow palate; short, broad hands; a single, transverse palmar crease; dysplasia of the midphalanx of the fifth finger; increased space between the first and second toes (sandal toes); and hyperflexible joints.

A 4-year-old boy with no previous medical care presents to a pediatrician for his first medical visit. His legal guardian states the boy's birth mother was 38 years of age when she gave birth to him at a refugee clinic. The patient is noted to have mild intellectual disability, normal language comprehension, and delayed language production. Physical examination findings are significant for upslanting palpebral fissures, prominent epicanthic folds, brachycephaly, and a flat facial profile. Which of the following additional physical examination findings is most likely to be present in this patient? ADigital transverse crease distinctly separate from the proximal transverse crease BExcess skin at the nape of the neck CRocker bottom feet DShield chest

AAlbendazole Permethrin (C) is the appropriate treatment for pediculosis and is not indicated for patients with pinworms. Treatment option includes mebendazole, albendazole, or pyrantel pamoate

A 4-year-old girl presents to her pediatrician with concerns of perianal itching. Her mother states her daughter has not been able to sleep at night because she is constantly scratching her anal area. On physical exam, there are excoriations surrounding the patient's anus. A tape test was performed, and bean-shaped structures were noted under the microscope. Which of the following is the most appropriate treatment for the suspected diagnosis? AAlbendazole BIvermectin CPermethrin DPiperazine

CPoststreptococcal glomerulonephritis rare kidney disease that is caused by exposure to group A beta-hemolytic Streptococcus. Other symptoms might include, fever, abdominal pain, and decreased urine output. Urine can look dark and cola-colored due to red blood cells present and frothy from protein levels within the nephrotic range. Diagnosis of poststreptococcal glomerulonephritis is made with either positive Streptococcus culture or elevated antistreptolysin O titers and clinical symptoms such as described above. Treatment is often supportive and might include antibiotic therapy if Streptococcus infection is still present.

A 4-year-old girl presents to the ED after telling her mother her urine looked funny. Upon examination, the child has generalized edema but no other specific findings. Her mother states the patient had a sore throat about 2 weeks ago and was treated with an antibiotic by her primary care physician. Vital signs include T 100.5°F, HR 110 beats per minute, RR 22 breaths per minute, and BP 125/95 mm Hg. Urinalysis is positive for red blood cell casts and protein. A throat swab and serology testing are pending. What is the most likely diagnosis based on the clinical findings? AAcute tubular necrosis BNephrotic syndrome CPoststreptococcal glomerulonephritis DUrinary tract infection

HSV symmetric papular <3 cm, target-like lesions

MCC of erythema multiform

pituitary tumor

MCC of growth hormone deficiency?

bicuspid aortic valve causing aortic stenosis

MCC of harsh crescendo-decrescendo systolic ejection murmur is auscultated, and an ejection click is auscultated at the apex. An increased left ventricular impulse is noted on chest palpation in pediatric patients?

DStaphylococcus aureus Bacterial tracheitis is a rare condition that typically affects pediatric patients, most commonly under the age of 6 but the mean age is 5 through 7 years, and is caused by a bacterial infection of the trachea. Patients will typically present with a history of cold-like symptoms followed by a rapidly progressive sore throat, cough, fever, difficulty breathing, and hoarseness. This history, along with audible stridor (which is a high-pitched sound with respirations), respiratory distress, and a toxic appearance, is highly suggestive of bacterial tracheitis. The clinical presentation of croup and bacterial tracheitis are similar, but patients with croup usually do not appear toxic and have a normal respiratory rate and low-grade fever. Hence, parainfluenza virus is less likely the cause of this patient's symptoms.

A 4-year-old girl presents to the emergency department with a cough, rhinorrhea, and a fever for 5 days. The patient's father states her symptoms were initially mild, but today the patient started to have difficulty breathing and a rapidly progressive fever. Vital signs reveal HR of 125 bpm, RR of 38 breaths per minute, BP of 110/72 mm Hg, T of 103.2°F, and oxygen saturation of 94%. The patient appears toxic on the exam. A high-pitched sound with both inspiration and expiration, use of accessory respiratory muscles, and a barking cough are notable. The patient is noted to have hoarseness but is tolerating oral secretions. The father confirms she is up to date on her vaccinations. Which of the following is the most common causative agent for the suspected diagnosis? AHaemophilus influenzae BParainfluenza virus CRespiratory syncytial virus DStaphylococcus aureus

BCefuroxime 1st gen ceph TOC in kiddos A second or third-generation cephalosporin (e.g., cefuroxime-2nd) is preferred and has a lower probability of resistance compared to trimethoprim-sulfamethoxazole and amoxicillin, which tend to have relatively high local resistance rates. Nitrofurantoin is also an option but is not preferred due to more frequent dosing (4 times a day) and increased risk of gastrointestinal symptoms.

A 4-year-old girl presents to the office with abdominal pain and a painful sensation while urinating for the past 2 days. She has never had these symptoms before. On exam, she is non-toxic appearing, afebrile, and has some tenderness to palpation over the suprapubic area. A urinalysis is positive for white blood cells, and the urine culture reveals the presence of more than 100,000 CFU/mL of Escherichia coli. Which of the following medications is indicated for the treatment of uncomplicated cystitis? AAmoxicillin BCefuroxime CNitrofurantoin DTrimethoprim-sulfamethoxazole

DRight ventricular outflow tract obstruction Tetralogy of Fallot is the most common cyanotic congenital heart condition and is comprised of right ventricular hypertrophy, a ventricular septal defect, an aorta that overrides the ventricular septum, and right ventricular outflow tract obstruction. While a boot-shaped heart on chest radiography and findings of right ventricular hypertrophy with right axis deviation on ECG suggest a diagnosis of tetralogy of Fallot, echocardiography is required to confirm the diagnosis and investigate the defect along with coronary and pulmonary artery anatomy.

A 5-week-old girl presents to the emergency department after her parents witnessed an episode of sudden-onset cyanosis with altered consciousness. Physical examination reveals a rough, crescendo-decrescendo systolic ejection murmur that is heard best at the left sternal border in the third intercostal space with radiation to the back. Which of the following defects is found in this condition? AAtrial septal defect BBicuspid aortic valve CLeft ventricular hypertrophy DRight ventricular outflow tract obstruction

D 9 weeks Colic is defined as an otherwise healthy infant who is < 3 months of age who cries for ≥ 3 hours per day ≥ 3 days per week without a clear explanation, such as hunger or a soiled diaper.

A 5-week-old term infant presents to the clinic with his mother, who reports he cries 4 hours per day without a clear explanation. The infant is gaining weight and feeding appropriately, and physical examination is unremarkable. By which of the following ages does this condition typically resolve? A12 weeks B24 weeks C6 weeks D9 weeks

Intussusception

MCC of intestinal obstruction in ages 6 mos- 36 mos

Psudomonas aeruginosa and tx w cipro/hydrocortisone drops

MCC of otitis externa and tx

AAvascular necrosis of the proximal femoral epiphysis Legg-Calvé-Perthes disease is idiopathic avascular necrosis of the proximal femoral epiphysis, which is likely due to an interruption of blood supply to the capital femoral epiphysis. It is most prevalent in children aged 3 to 12 years (with a mean age of 5 to 7 years), and it affects boys more than girls at a 4:1 ratio. Risk factors include obesity, skeletal immaturity, low socioeconomic status, and hypercoagulable state (factor V Leiden). radiates to the ipsilateral groin, thigh, or knee. A physical exam reveals decreased internal rotation and abduction with some discomfort. A plain hip X-ray (anterior and frog-leg views) may be normal in early disease but may show fragmentation and then healing of the femoral head, with residual deformity.

A 5-year-old boy presents with an insidious onset of a limp and right hip pain that radiates to the right groin and thigh for 4 months. He reports worsening pain with activity that is not always relieved by rest or pain medication. He reports no previous injury. On exam, you note decreased right hip internal rotation and abduction with some discomfort. There is also mild atrophy of the anterior aspect of the right thigh without erythema, warmth, or effusion on the right hip joint. A plain radiograph of the right hip demonstrates fragmentation and healing of the right femoral head. Which of the following is the most likely cause of his symptoms? AAvascular necrosis of the proximal femoral epiphysis BOsteoid osteoma of the proximal femoral epiphysis CSeptic arthritis of the proximal femoral epiphysis DSlippage of the proximal femoral epiphysis

AHand-gripping Patients with a small ventricular septal defect are often asymptomatic, with only a harsh, holosystolic murmur and palpable thrill on physical exam. Maneuvers that increase peripheral vascular resistance will increase the murmur of a ventricular septal defect, as the left ventricle will pump harder against the peripheral resistance and thus pump blood with greater velocity through the septal defect. Hand-gripping and squatting increase peripheral vascular resistance and will increase a ventricular septal defect murmur.

A 5-year-old girl presents to the clinic as a new patient for an annual checkup. Her vital signs are within normal limits, and her caretaker has no concerns. On physical exam, a harsh holosystolic murmur is auscultated at the mid-left sternal border, and a thrill is palpated at the third left intercostal space. Which of the following maneuvers is likely to increase the intensity of this child's murmur? AHand-gripping BLying in the left lateral decubitus position CStanding abruptly DValsalva maneuver

BMast cell release of histamine in the superficial dermis Urticaria, also known as hives or wheals, is a dermatologic reaction characterized by intense pruritus, erythematous plaques, and occasionally, angioedema. Urticaria is an allergic reaction mediated by cutaneous mast cells and basophils in the superficial dermis. These cells release histamine and vasodilatory mediators, resulting in pruritus and localized edema.

A 5-year-old girl presents to the urgent care clinic with her father. The father is concerned about an enlarging rash on the patient's arm. He states the patient spent the night in a tent in their backyard last night, and this morning, she woke up with a small lesion on her left arm that was very itchy. Over the last 2 hours, the lesion has progressively enlarged. Physical examination of the left arm reveals an erythematous, raised, circular plaque that is 6 cm in diameter. The rest of her physical examination is normal. The patient is prescribed cetirizine, and a follow-up call the next morning reveals the lesion has resolved entirely. Which of the following best describes the pathophysiology of the patient's condition? AEosinophil release of vasodilatory mediators in the subcutaneous tissues BMast cell release of histamine in the superficial dermis CMonocyte release of interleukin 6 in the bloodstream DNeutrophil release of specific granules containing lactoferrin in the bloodstream

AHaemophilus influenzae type b The most common cause of epiglottitis in a child is Haemophilus influenzae type b. Due to the initiation of the Hib vaccine, the incidence of epiglottitis is declining. The patient will typically present with the three Ds: dysphagia, drooling, and respiratory distress. Tropoding, muffled voice, inspiratory stridor, retractions, fever, and a sore throat. Thumbprint sign on Xray***

A 5-year-old unvaccinated girl presents to the emergency department with her mother, who states she has had difficulty swallowing and trouble breathing for the past few hours. Her mother notes she had a fever and sore throat since yesterday evening. On exam, you note the patient is sitting on her mother's lap and leaning forward with her neck hyperextended. She appears to be uncomfortable, with audible inspiratory stridor on exam and intercostal retractions. Her vitals are as follows: heart rate 120 bpm, respiratory rate 30/min, oxygen saturation 90% on room air, and blood pressure 110/65 mm Hg. Which of the following organisms is the most common cause of this patient's suspected diagnosis? AHaemophilus influenzae type b BParainfluenza virus CStaphylococcus aureus DStreptococcus pyogenes

DSupplement vitamins A, D, E, and K in addition to pancreatic enzymes (d/t pancreatic insuff common in CF) steatorrhea, bulky foul-smelling stools, meconium ileus at birth, low birth weight, and infertility. The steatorrhea and bulky, pale, foul-smelling stools are an indication of pancreatic insufficiency in a cystic fibrosis patient. In addition to pancreatic enzyme replacement therapy, it is important to supplement fat-soluble vitamins A, D, E, and K to prevent vitamin deficiencies.

A 6-month-old boy presents to the clinic due to weight loss, steatorrhea, and increased frequency of bulky, pale, and foul-smelling stools. Past medical history is significant for meconium ileus at birth and a positive sweat chloride test performed at 3 weeks of age. What is the best initial clinical intervention based on the suspected diagnosis and his current symptoms? AAdminister airway clearance therapies BInitiate docusate sodium CInitiate loperamide DSupplement vitamins A, D, E, and K

ARolls front to back

A 6-month-old girl presents to the clinic for a well-child check. She was born at 39 weeks gestation via uncomplicated vaginal delivery and has had no medical conditions arise since birth. She is exclusively breastfeeding and is appropriately gaining weight. Which of the following milestones is appropriate for this patient? ARolls front to back BSays "mama" nonspecifically CSits without support DTransfers toys hand to hand

C Rubeola aka measles **3 C's cough, coryza, conjunctivitis coryza inflam of oral mucosa **koplik spots!! Rash spreads from face down **mmr vaccine

A 6-month-old girl presents to the office with a rash for the last 24 hours. Her mother states that the patient had a fever, conjunctivitis, and cough for 2 days prior to the rash and continues to have these symptoms. The rash started on her face and then moved to her belly and extremities. The patient's temperature in the office is 101°F. On physical exam, you note a red, maculopapular rash over the face, trunk, and extremities that does not blanch with pressure. In her mouth, you note small, irregular red spots with a white center on the buccal mucosa opposite the molars bilaterally and moderate pharyngeal erythema. Which of the following is the most likely diagnosis? ARoseola BRubella CRubeola DScarlet fever

BConstipation Constipation is secondary to both decreased smooth muscle tone and autonomic dysfunction. Other gastrointestinal manifestations include nausea and anorexia. Hypercalcemia Sx: bone pain (bones), kidney stones (stones), abdominal pain (groans), lethargy, psychosis/depression (psychiatric overtones) Altered mental status (A), including confusion and lethargy, can be seen in patients with severe hypercalcemia, noted as calcium levels > 14 mg/dL.

A 6-year-old boy is diagnosed with acute lymphocytic leukemia and presents with bone pain. A serum calcium level is 11.2 mg/dL, and an intact parathyroid hormone level is 8 pg/mL. Which of the following would you expect to find while taking a history and performing a physical exam, given these lab abnormalities? AAltered mental status BConstipation CPolyuria DProfound muscle weakness

DViral conjunctivitis MCC adenovirus, Patients usually present with a red eye, watery or mucoserous discharge, and a burning, sandy, or gritty feeling in one eye. A physical exam may show profuse tearing, follicular- or bumpy-appearing tarsal conjunctiva, and an enlarged, tender preauricular lymph node. tx: supportive care, with the exception of HSV conjunctivitis, which requires topical (ganciclovir 0.15%) and systemic antiviral (acyclovir, valacyclovir).

A 6-year-old boy presents with his father, who states the child has been experiencing bilateral eye redness, watery discharge, and low-grade fever for 3 days. He also reports morning crusting followed by daytime watery discharge. On exam, you note follicular-appearing tarsal conjunctiva, bilateral conjunctival erythema, watery discharge, and tender preauricular lymphadenopathy. Which of the following is the most likely diagnosis? AAllergic conjunctivitis BBacterial conjunctivitis CBlepharitis DViral conjunctivitis

AAlarm therapy Enuresis refers to involuntary urination that can occur at any time of the day. Bedwetting is a term that typically refers to enuresis that occurs at night. A urinalysis is the most important screening test in a child with involuntary voiding, with a culture and sensitivity ordered if the results are positive for possible infection. Initial treatment should be focused on behavioral modification with positive reinforcement. If the patient continues to have episodes of enuresis after 3 months, other therapies, such as alarm therapy and medications, have been shown to be helpful. Desmopressin medication of choice in children with enuresis. However, first-line therapy should either be with alarm therapy or desmopressin, rather than both.

A 6-year-old boy presents with his mother with reports of frequent bedwetting for the past 2 months. He does not remember voiding and has no pain with urination. He has normal bowel movements. His mother explains that she and the patient's father recently got divorced, and she thinks this may be related to the boy's symptoms. A urinalysis dipstick is negative. When discussing treatment options, the patient's mother states she would rather use a long-term treatment strategy with low relapse rates. After behavioral modification, which of the following treatments should be recommended for this patient? AAlarm therapy BBladder training exercises CDesmopressin DOxybutynin

CInsufficient intake of green vegetables and citrus fruits Vitamin C deficiency, also known as scurvy, is the result of insufficient dietary intake of green vegetables and citrus fruits. Symptoms of vitamin C deficiency include perifollicular hyperkeratosis and corkscrew hairs, hemorrhages (e.g., petechiae, gingivitis, hemarthrosis, impaired wound healing), fatigue, malaise, irritability, and glossitis.

A 6-year-old boy with a history of autism presents to the clinic with a 2-week history of excessive fatigue, malaise, easy bruising, and bleeding from his gums. Vital signs show BP of 100/60 mm Hg, HR of 85 bpm, and T of 98.4°F. Exam findings include a red swollen tongue, skin pallor, and a raised papular rash with corkscrew hairs on the arms and thighs. Which of the following dietary histories is most likely the cause of the patient's symptoms? AA predominantly corn-based diet BInsufficient intake of animal-based foods CInsufficient intake of green vegetables and citrus fruits DInsufficient intake of vegetables rich in beta-carotene, dairy, eggs, and meat

DProvide reassurance and supportive treatment The exanthem associated with hand, foot, and mouth disease may be composed of macular, maculopapular, and vesicular lesions. palms, soles, heels, mouth!!!***

A 6-year-old girl presents to her pediatrician's office with a sore throat and a nonpruritic, painless rash that developed suddenly 2 days ago. She was previously healthy and is up to date on all of her immunizations. Her vital signs are T of 37.9°C (100.2°F), BP of 98/74 mm Hg, HR of 85 bpm, RR of 18 breaths per minute, and oxygen saturation of 99% on room air. Physical examination reveals an alert and oriented, well-appearing child with vesicular lesions on the tongue and buccal mucosa surrounded by an erythematous rim. No tonsillar exudates or lymphadenopathy are appreciated. A maculopapular rash is present over the palms, interdigital web spaces, dorsum of the toes, soles, and heels. A rapid strep test is negative. Which of the following is the most appropriate clinical intervention at this time? AAdminister amoxicillin BOrder antistreptolysin O antibody titer CPerform skin scraping of the interdigital spaces and microscopic examination DProvide reassurance and supportive treatment

B. Aortic 3 sign Coarctation of the aorta refers to a narrowing of the aorta just distal to the left subclavian artery. increased blood pressure in the upper extremities and decreased or unreadable blood pressure in the lower extremities. Pulses may be present in the lower extremities but are diminished in severe disease. There is also a pulse lag between upper extremities and lower extremities (brachial-femoral pulse delay). Xray: posterior rib notching & aortic 3 sign can be visualized. 3 sign: formed from a round superior portion of the aortic bulb, a tight indentation of the aortic wall at the site of coarctation, and then a dilatation of the aorta distal to the stricture. DX: Echo TX: balloon angioplasty or open surgical repair.

A 6-year-old girl presents to the clinic for an annual exam. She has no serious health concerns but states sometimes her legs hurt during recess and feel better when she rests. On physical exam, the child is well nourished and well developed, and vital signs are within normal limits, except for a blood pressure of 140/80 mm Hg. Her pedal pulses are slightly diminished bilaterally. When both the radial and femoral pulses are palpated simultaneously, there is a marked delay in the femoral pulses. Chest auscultation reveals a systolic murmur that extends beyond the second heart sound and is heard best at the left paravertebral interscapular area. Which of the following common radiographic findings in patients with this child's presumed pathology is demonstrated in the X-ray above? A. Anterior rib notching B. Aortic 3 sign C. Boot-shaped heart D. Kerley B-lines

ACongenital abnormality Wilms tumor is a common kidney malignancy found in children, with most occurring before the age of 10. Wilms tumors often present with abdominal pain and a hard, round, smooth, nontender mass that does not cross the midline. Wilms tumors are associated with many congenital abnormalities, and these should be considered when a Wilms tumor is found in a child. Some of the most common congenital abnormalities include Wilms tumor, aniridia, genitourinary anomalies, and intellectual disability (originally mental retardation) (WAGR) syndrome, Denys-Drash syndrome, and Beckwith-Wiedemann syndrome.

A 7-year-old boy presents to the clinic due to abdominal pain. He reports no nausea, vomiting, changes in diet, or changes in bowel movements. Vital signs include T 100°F, HR 110 bpm, RR 22 breaths per minute, and BP 130/95 mm Hg. Upon exam, there is a large, smooth, nontender, firm mass located in the left lower quadrant. CT imaging is shown above. Which of the following is most likely to be found during history-taking based on the suspected diagnosis? ACongenital abnormality BHistory of intussusception CPremature birth DUrinary tract obstruction

DSuperficial partial-thickness burn This patient has a superficial partial-thickness burn, which involves the epidermis and superficial dermis. These burns form blisters within 24 hours and are painful. The skin appears red and weeping and blanches with pressure. Treatment is cooling the skin, relieving pain, and debriding blisters if they rupture. Superficial partial-thickness burns typically heal with minimal scarring. Burn wounds often have multiple levels of injury and may evolve with increasing tissue damage over several days, so it is important to arrange for follow-up and monitor patients with burn injuries.

A 6-year-old girl presents to your office with her parents for follow-up of a burn injury to her right forearm that occurred 48 hours ago. She bumped the arm against a wood stove while playing at her friend's house and was seen in the emergency department immediately. The burn was cleaned, and her parents were instructed to apply cold compresses and give her acetaminophen for pain. On physical examination, she has an area of erythema over her lateral upper forearm, about 4 cm in diameter. There is a blister leaking a small amount of serous fluid. The area is extremely painful and blanches with pressure. Which of the following best describes the patient's injury? ADeep partial-thickness burn BFull-thickness burn CSuperficial burn DSuperficial partial-thickness burn

DFrequently changing the diaper and leaving it off for a few hours each day This patient has a presentation consistent with mild to moderate diaper dermatitis. It is a common condition due to disruption of the skin barrier, which allows access to chemical irritants and microorganisms. Moisture and friction from the diaper lying against the skin, increased pH from fecal bacteria and urine, and enzymes in feces all contribute to this breakdown. Risk factors include diarrhea, broad-spectrum antibiotic use (which predisposes to diarrhea), and feeding with formula, as breast-fed infants have a lower stool pH. Mild diaper dermatitis presents with mild erythema, scattered papules, and minimal maceration. Moderate dermatitis presents with pain and more extensive erythema with maceration and superficial erosions. sparing folds

A 7-month-old girl presents to the office with her parents, who are concerned about recurrent diaper rashes. They have noticed more skin irritation recently, and she sometimes cries during diaper changes. They recently started giving her solid foods, and she has had some loose stools. She is otherwise healthy and is on track with her developmental milestones. Physical examination reveals erythematous papules over the buttocks, perineum, and upper thighs with sparing of the skin folds. There is mild maceration without erosion of the skin. The dermatitis is localized to the diaper area, and findings on the remainder of her skin examination are normal. Which of the following is the most appropriate initial treatment? AApplying baby powder or cornstarch to the diaper area BApplying triple antibiotic ointment to the affected areas CCleaning the diaper area with baby wipes rather than tap water DFrequently changing the diaper and leaving it off for a few hours each day

DPrednisone Idiopathic nephrotic syndrome is a kidney disease that causes increased permeability across the basement membrane, leading to severe proteinuria, albuminuria, hyperlipidemia, and edema. Minimal change disease is the most common type of nephrotic syndrome in children. Typical symptoms include peripheral, periorbital, and scrotal edema. Urinalysis includes oval fat bodies and proteinuria, which prompts the need for a 24-hour urine protein collection to make the diagnosis. Treatment is first done with corticosteroids, as steroid responsiveness is high in the majority of patients. A titrated dose of prednisone is often the first-line medication.

A 7-year-old boy presents to the clinic due to severe edema, as shown in the image above. His father states there is also swelling in his scrotum, and his lower extremities seem to be swollen as well. Upon examination, there is notable pitting in his lower extremities. Urinalysis shows proteinuria and oval fat bodies. A 24-hour urine protein collection is initiated. What is the best initial treatment for this condition based on the suspected diagnosis? ACyclobenzaprine BCyclophosphamide CPenicillin DPrednisone

A> 30% of body surface area involved Toxic epidermal necrolysis is a life-threatening mucocutaneous disease most commonly precipitated by medication use. sulfa drugs, anticonvulsants, NSAIDs. High risk HIV, autoimmune, malignancy, genetics. Toxic epidermal necrolysis exhibits Nikolsky sign, which is epidermal detachment secondary to gentle lateral pressure. Toxic epidermal necrolysis and Stevens-Johnson syndrome are differentiated by the percent of total body surface area involved, with the former including > 30% and the latter including < 10%.

A 7-year-old boy presents to the urgent care clinic for evaluation of the rash over his body. On physical exam, there is blistering and large areas of peeling skin. The patient's mother states he recently began treatment for epilepsy. Physical evaluation further reveals lesions on the oral mucosa, and sloughing of the skin is noted with gentle lateral pressure. Which of the following is likely additionally noted on history and physical exam findings? A> 30% of body surface area involved BInitial appearance of lesions on the distal extremities CInvolvement of the palms and soles DRecent Staphylococcus aureus infection

CSystemic juvenile idiopathic arthritis intermittent high fevers, arthritis, and a rash. Fever is often the presenting symptom, typically with a daily spike (quotidian pattern) and spontaneous return to normal. arthritis is typically oligoarticular or polyarticular and can involve any joint, most often the wrists, knees, and ankles. The joints may be swollen and tender, but the child can typically bear weight. The rash is pink, macular, and may be anywhere on the body but is often over the torso and may be pruritic. The rash is intermittent and is most prominent during the fever spikes. Other findings include lymphadenopathy and hepatosplenomegaly, pericarditis, and pulmonary involvement, especially pleural effusions. Tx: NSAIDS, MTX, steroids, DMARDS refer**

A 7-year-old boy presents to your office with his parents, who note he has been limping intermittently for the past 2 months and has been less active. His symptoms seem to be worse in the morning and improve once he starts walking. More recently, he has had several episodes of being ill at school, and yesterday he was sent home with a fever of 39°C. He also had a rash over his torso. He was given acetaminophen, and both the fever and rash resolved. He reports feeling well this morning. On physical examination, he is a healthy-appearing boy. He is afebrile. He walks with a mild stiff-knee gait. He does not have any skin abnormalities. He has mild effusions in both knees and has mild restriction of range of motion, measured at 0° to 120° bilaterally. He has no pain with hip range of motion. He has mild swelling of the right wrist and has 45° of wrist dorsiflexion compared to almost 90° on the left. The remainder of his examination is unremarkable. Laboratory tests reveal a hematocrit of 30%, white blood cell count of 20,000 cells/mm3, platelet count of 650,000/mm3, and erythrocyte sedimentation rate of 90 mm/hr. Antinuclear antibody and rheumatoid factor tests are negative. Urinalysis is negative for hematuria or proteinuria. Which of the following is the most likely diagnosis? A Acute leukemia B Septic bacterial arthritis C Systemic juvenile idiopathic arthritis D Transient synovitis

BCoronary artery abnormalities MC serious complication of Kawasaki dz!!!***including dilation, aneurysm, and stenosis.

A 7-year-old boy presents with fever and bilateral conjunctivitis for 5 days. Vital signs are temperature of 38.4°C, respiratory rate of 22 bpm, heart rate of 105 bpm, blood pressure of 90/55 mm Hg, and oxygen saturation of 98%. On exam, you note left cervical lymphadenopathy, bilateral hand swelling and erythema, oral mucositis, and a generalized erythematous macular rash. Laboratory studies demonstrate leukocytosis with predominant immature neutrophils and elevated CRP and ESR. Which of the following is the most common complication of the suspected diagnosis? AAcute kidney injury BCoronary artery abnormalities CHydrops of the gallbladder DSensorineural hearing loss

DPreseptal cellulitis Orbital cellulitis (C) may present similarly to preseptal cellulitis but typically has additional alarm signs on exam, including pain with extraocular movements, proptosis, and decreased visual acuity. Preseptal cellulitis (also referred to as periorbital cellulitis) is an infection of the soft tissues anterior to the orbital septum. It is most often caused by infection from surrounding tissues of the face and eyelids following trauma, insect bites, animal bites, or foreign bodies. More rarely, infection can arise from sinusitis. Patients with preseptal cellulitis typically report unilateral ocular pain with eyelid swelling and redness. Most importantly, preseptal cellulitis must be distinguished from orbital cellulitis, which includes infection of ocular muscles and fat posterior to the orbital septum, and is threatening to the patient's vision and life.

A 7-year-old girl presents to the clinic with a 2-day history of swelling and redness of her left upper eyelid. Physical exam reveals erythema and edema of the left upper eyelid and periorbital tissue, a mild skin abrasion near her left eyebrow, and visual acuity that is within normal limits. The patient reports no pain with eye movements or changes in her vision. What is the most likely diagnosis? AAcute conjunctivitis BHordeolum COrbital cellulitis DPreseptal cellulitis

DWeak femoral pulses Coarctation of the aorta is an acyanotic congenital heart defect characterized by narrowing in the aortic arch. Chest radiography reveals notching of the ribs and a figure 3 sign with an indent at the level of the coarctation and dilation of the pre- and poststenotic segments.***

A systolic murmur with an apical ejection click is noted in the left axilla with radiation to the interscapular area of a 5-year-old boy. Chest radiography findings are shown above. Which of the following is a hallmark physical examination finding? APulsus alternans BPulsus paradoxus CWater hammer pulse DWeak femoral pulses

DSeverity of retinal hemorrhages is associated with the likelihood of abuse they typically begin to resolve within 24 to 48 hours and clear within 1 to 2 weeks. The severity correlates with both the extent of intracranial injury and neurologic impairment.

A 9-month-old girl presents to the emergency department with her mother, who states the infant would not wake up this morning when she tried to take her out of her crib. She states the infant was fine last night. On physical examination, the patient is of average size for her age. She is lethargic but breathing spontaneously. Her pupils are poorly reactive. Funduscopic examination reveals extensive bilateral retinal hemorrhages. Which of the following statements most accurately describes the significance of retinal hemorrhages in head trauma due to abuse? AIntraretinal hemorrhages may persist for up to 6 months BRetinal hemorrhages are present only in abusive head trauma CSeverity of retinal hemorrhages does not correlate with the degree of neurologic impairment DSeverity of retinal hemorrhages is associated with the likelihood of abuse

B Azithromycin Pertussis infections are caused by the bacteria Bordetella pertussis. This is a human pathogen with no known animal or environmental reservoir. It is spread through respiratory droplets from coughing, sneezing, or sharing breathing space for extended periods of time. The incubation period is typically 7-10 days. Patients typically present with paroxysms of cough, an inspiratory whoop, and, in young children and infants, posttussive vomiting. Classic pertussis is divided into three stages: the catarrhal stage, paroxysmal stage, and convalescent stage. The catarrhal stage is similar to a viral upper respiratory infection. Patients have a mild cough and coryza, and fever is not typically present. It generally lasts about 2 weeks. The paroxysmal stage is when the coughing spells increase in severity and are characterized by a long series of coughs with little to no inspiration.

A 9-month-old girl presents with a severe cough. Her mother states that she has had the cough for 2 weeks, but it has gotten worse over the past several days. She now reports that the patient has coughing episodes where she is having trouble breathing followed by vomiting. Which of the following is the recommended treatment given the most likely diagnosis? AAmoxicillin BAzithromycin CCeftriaxone DTrimethoprim-sulfamethoxazole

BPainful, polygonal papules Cutaneous lichen planus is a dermatologic condition that typically affects middle-aged adults 30-60 years of age. Children can be affected as well, but this is uncommon. There is also an association between lichen planus and hepatitis C virus five Ps: purple, pruritic, polygonal, papular, and plaques. Pain is rarely associated with lichen planus. Wickham striae, or fine white lines on the surface of the lesions, may be observed. he development of lesions at sites of trauma is known as the Koebner phenomenon, which can occur when patients frequently scratch these intensely pruritic lesions. Topical corticosteroids are considered first-line therapy. + antihistamines to help prurititis

A 9-year-old boy presents to his pediatrician's office with an intensely itchy rash. He states the rash started 2 weeks ago on his ankles and wrists. He reports no lesions on his trunk. No other family members have similar symptoms. Close examination of the ankles and wrists reveals multiple lesions with fine white lines on the surface. No other lesions are observed on the body. Which of the following findings on physical examination is most consistent with the suspected diagnosis? ACollarette scale BPainful, polygonal papules CS-shaped burrows DViolaceous lesions at sites of trauma

ABacterial rhinosinusitis Most cases of orbital cellulitis are caused by bacterial rhinosinusitis. The most common pathogens are Staphylococcus aureus and Streptococcus anginosus. Orbital cellulitis presents with eyelid swelling, eye pain with and without eye movement, and ophthalmoplegia. In some cases, there may also be vision impairment, chemosis, proptosis, and leukocytosis. The diagnosis of orbital cellulitis is confirmed by computed tomography scan, which would show inflammation of extraocular muscles, fat stranding, and likely anterior displacement of the eye globe. Most patients with orbital cellulitis can be treated with broad-spectrum antibiotics, such as vancomycin, in addition to a third-generation cephalosporin, such as ceftriaxone or cefotaxime. In rare cases, surgery may be needed

A 9-year-old girl presents to the emergency department accompanied by her mother, who notes the patient woke up this morning with significant redness and swelling to her right eye. On physical exam, the patient reports pain when moving her right eye, and the right eye is also tender to palpation, swollen, and erythematous. Which of the following is the most common cause of the suspected diagnosis? ABacterial rhinosinusitis BDacryocystitis CMiddle ear infection DOrbital trauma

AAmoxicillin-clavulanate >10 days suspect acute bacterial rhinosinusitis. Acute bacterial rhinosinusitis is believed to be caused by the accumulation of mucus in the sinus cavity due to edematous mucosa most commonly due to a viral upper respiratory infection that then becomes secondarily infected by bacteria. Antibiotics should be reserved for patients who have symptoms for more than 10 days, have severe symptoms, or are immunocompromised. First-line therapy is with amoxicillin-clavulanate.

A 9-year-old girl presents with cough and nasal congestion for the last 2 weeks. She states that her symptoms have not improved during this time. On physical exam, you note erythematous and edematous nasal turbinates with clear discharge and erythematous pharynx with clear postnasal drip. What is the recommended intervention for this patient? AAmoxicillin-clavulanate BMometasone furoate CObservation DSinus aspiration

CRubella Many cases are asymptomatic, but symptoms include a maculopapular rash 14 to 17 days postexposure with minimal systemic symptoms. Patients may have a low-grade fever concurrently or up to 5 days prior to the appearance of the rash. The rash appears first on the face and then spreads to the trunk and extremities within 24 hours.

A 9-year-old girl presents with her parents. They state that the patient has a rash that appeared yesterday. They state that the rash was initially only on her face but has since spread to her trunk and extremities. The patient is up-to-date on her vaccinations, and the family just returned from a visit to South Africa. Vital signs are within normal limits except for a temperature of 99.5°F. On physical exam, you note a well-developed, well-nourished, 9-year-old girl in no acute distress. You note a generalized maculopapular rash as well as posterior cervical and posterior auricular lymphadenopathy. When asked about sick contacts, the parents state that the friends they were staying with had a child about the same age who had a similar rash when they arrived in South Africa about 2 weeks ago. What is the most likely diagnosis? AParvovirus B19 BRoseola CRubella DScarlet fever

BHepatomegaly Congenital cytomegalovirus infection is a common neonatal infection and the leading cause of nonhereditary sensorineural hearing loss in newborns. Cytomegalovirus (CMV) is a member of the herpesvirus family. CMV is part of the congenital TORCH syndrome, The most common manifestations include petechiae, jaundice, hepatosplenomegaly, small size for gestational age, microcephaly, sensorineural hearing loss, lethargy, poor suck, chorioretinitis, seizures, hemolytic anemia, and pneumonia. Sensorineural hearing loss is a common sequela of congenital CMV and presents in up to one-half of infants with symptomatic disease. Treatment is indicated in patients with virologically confirmed CMV and involvement of at least one end-organ system and consists of antiviral medications including IV ganciclovir or valganciclovir.

A newborn infant is admitted to the neonatal intensive care unit due to premature birth at 34 weeks gestation, low birth weight, and rash. Exam reveals diffuse purpura and petechiae. Severe thrombocytopenia is noted on complete blood count. Which of the following additional findings is most likely to be present on the physical exam? ACyanosis BHepatomegaly CHypotonia DMacrocephaly

DHeterophile antibody test Epstein-Barr virus (EBV) is the viral pathogen associated with infectious mononucleosis.The classical symptoms of infectious mononucleosis are low-grade fever, fatigue, malaise, lymphadenopathy, and pharyngitis lasting 2-3 weeks. A smaller percentage of patients will also present with a generalized maculopapular rash. On exam, pharyngeal inflammation and tonsillar exudates are often noted. Lymph node involvement is usually symmetric with some tenderness, most notably on the posterior cervical and auricular nodes. An abdominal exam should be done to evaluate for splenomegaly, an important risk factor for splenic rupture, which is one of the more concerning potential complications of EBV. The primary diagnostic test for symptomatic patients with suspected EBV is the heterophile antibody, or mononucleosis spot, test. refrain from contact sports for at least 4 weeks

A previously healthy 12-year-old boy presents to his pediatrician with fever, fatigue, and sore throat for 3 days. His maximum temperature at home has been between 100 and 101°F. On exam, he looks tired but nontoxic. Both his tonsils are enlarged and covered with a white exudate. His cervical lymph nodes are also enlarged, most prominently on the back of his neck. He has no rash, but there are some palatal petechiae noted. A throat swab is done, and the polymerase chain reaction streptococcal swab is negative. Which of the following is the best next test for this patient based on concern for a viral process? AComplete blood count BEpstein-Barr virus DNA quantification CEpstein-Barr virus-specific antibody testing DHeterophile antibody test

AFull-thickness keratinocyte epidermal necrosis Stevens-Johnson syndrome (SJS) is part of a continuum of serious mucocutaneous reactions characterized by epidermal detachment and necrosis. When > 30% of the body surface area is affected, the disease is then termed toxic epidermal necrolysis (TEN). triggers, medications (most common) and infections. Diagnosis is typically clinical, and clinicians should attempt to elicit any history of preceding drug exposure or febrile illness. For a definitive diagnosis, a skin biopsy will reveal partial to full-thickness keratinocyte necrosis of the epidermis.

A previously healthy 4-year-old girl presents to the ED with fever, concerning skin changes, and pain. Other than fever and tachycardia, her vital signs are unremarkable. On exam, she appears ill, with obvious erythema over her face down to her shoulders. Her eyes are red and watery, and her lips are cracked and bleeding. A few vesicles can be seen around her mouth and over the skin on her neck. Examination of the skin demonstrates positive Nikolsky sign. An estimated 9% of her body surface area is affected. She is extremely agitated, screams with exam, and is difficult to console. Her parents report she was recently diagnosed with a urinary tract infection and was started on trimethoprim-sulfamethoxazole, which she had not taken previously. She completed the 7-day course today. She is admitted for additional workup and supportive care. Which of the following found on skin biopsy supports the suspected diagnosis? AFull-thickness keratinocyte epidermal necrosis BInterface dermatitis with inflammatory infiltrate of lymphocytes, neutrophils, and eosinophils CSpongiform, subcorneal, or intraepidermal pustules DSubcorneal cleavage in the epidermis

C Intravenous nafcillin Staphylococcal scalded skin syndrome (SSSS) is a toxin-mediated bacterial skin disorder that primarily affects young children aged 0-6 years. Certain strains of Staphylococcus aureus produce exotoxins and, when disseminated to the skin, can lead to diffuse skin pain, erythema, superficial blisters, and desquamation. Unlike Stevens-Johnson syndrome (SJS) and toxic epidermal necrolysis (TEN), mucous membranes are typically not involved. Untreated, SSSS can be complicated by skin loss, hypovolemia, electrolyte imbalance, secondary infection, septicemia, and death.

A previously healthy 5-year-old boy presents to the ED with a rash that looks and feels like a sunburn, though he has not been in the sun. It started on the back of his neck and has now spread all over his trunk and face within the last few hours. A few areas are starting to blister. His parents report he was fussier than usual yesterday, had a low-grade fever, and did not eat well, but the rash did not appear until today. He did not improve with acetaminophen or ibuprofen and has not received any new medicines. On exam, he appears miserable but in no acute distress as long as he is not being touched. A thorough skin exam is notable for diffuse symmetrical erythema with a sunburned appearance over his neck down to his trunk, including his axillae and gluteal cleft. A few areas with flaccid bullae are present, as well as some desquamating bullae on his neck and upper back. He has crusting around his mouth and nose with a dried oatmeal appearance but no sloughing, cracking, or bleeding of his mucous membranes. A nasal swab and weeping areas of the crusted perioral lesions are sent for culture to assist with his diagnosis. Which of the following is the recommended empiric first-line treatment? AIntravenous clindamycin BIntravenous immune globulinYour Answer CIntravenous nafcillin DIntravenous vancomycin

ADexamethasone Respiratory distress syndrome, previously known as hyaline membrane disease, is a pulmonary disorder in preterm infants caused by deficient production and inadequate activation of pulmonary surfactant.

A woman who is at 30 weeks gestation presents to her obstetrician for a routine follow-up appointment. She reports no new symptoms at this time. Vitals indicate HR 92 bpm, RR 16 breaths per minute, BP 172/98 mm Hg, T 98.8°F, and SpO2 99% on room air. Urine dipstick analysis reveals 2 proteinuria. Which of the following medications should be administered to reduce fetus mortality and morbidity? ADexamethasone BHydrocortisone CMannitol DPhenobarbital

C Rubella While mumps, roseola, rubella, and rubeola can all cause exanthems, rubella is the only one that is teratogenic. Maternal infection with rubella virus can result in congenital rubella infection, which may result in fetal death, preterm delivery, or congenital rubella syndrome.

Which of the following exanthema is teratogenic in the first trimester and may result in fetal hearing impairment, infantile glaucoma, and cardiac disease? AMumps BRoseola CRubella DRubeola

BAutosomal recessive Cystic fibrosis is a genetically inherited disorder that results in the acquisition of a mutated gene for the cystic fibrosis transmembrane conductance regulator (CFTR) protein on chromosome 7.

Which of the following inheritance patterns best describes the transmission of CF? AAutosomal dominant BAutosomal recessive CX-linked dominant DX-linked recessive

CRight atrial hypertrophy Atrial septal defect is a congenital heart defect in which blood flows from the high-pressure oxygenated left atrium to the lower-pressure deoxygenated right atrium through an atrial wall defect. This pathologic shunting of blood initially results in right atrial hypertrophy. wide, fixed split of the second heart sound (S2).

An 18-month-old boy presents to the clinic for his routine wellness exam. He has previously had recurrent respiratory infections and was diagnosed in early infancy with failure to thrive. Vitals today indicate HR 128 bpm, RR 18 breaths per minute, T 99.1°F, and SpO2 99% on room air. Cardiac auscultation reveals a wide, fixed, split S2 that is persistent in both the supine and standing position. Which of the following is a common pathophysiologic sequela of this congenital heart defect? ALeft ventricular hypertrophy BPulmonic stenosis CRight atrial hypertrophy DVentricular dysrhythmia

AAmoxicillin Scarlet fever (scarlatina) is a diffuse erythematous eruption that typically occurs as a complication of streptococcal pharyngitis. The scarlet fever rash typically starts in the groin and armpits before expanding to cover the trunk and then the extremities. Circumoral pallor, strawberry tongue (red and bumpy), and Pastia lines (red and linear petechial rash in the antecubital fossae and axillae) are other classic features.

An 8-year-old boy presents to the clinic with a diffuse erythematous rash. On exam, you notice circumoral pallor, strawberry tongue, and erythematous petechial lines in his antecubital fossae. Which of the following is the recommended first-line treatment for the most likely diagnosis? AAmoxicillin BDoxycycline CSupportive care

BPeritonsillar abscess This is often caused by Streptococcus pyogenes or is polymicrobial. Patients with a peritonsillar abscess commonly present with a severe unilateral sore throat, fever, and a hot potato voice. They may also be drooling, or have pooling of saliva, and trismus. On physical exam, they may have unilateral neck swelling and uvula deviation to the contralateral side. The tonsils will be enlarged and fluctuant. Treatment is with antibiotics, such as ampicillin-sulbactam (unasyn) or clindamycin.

An 8-year-old boy presents to the emergency department with his father, reporting progressing right-sided throat pain and a fever that began 7 days ago. When speaking to him, you notice his voice is muffled and he is uncomfortable but in no acute respiratory distress. On physical exam, he is drooling, his uvula is deviated toward the left, and he has significant swelling to the right side of his neck. Anterior cervical lymphadenopathy is also noted. Which of the following is the most likely diagnosis? AEpiglottitis BPeritonsillar abscess CRetropharyngeal abscess DStreptococcal pharyngitis

CTinea capitis Tinea capitis is a superficial fungal infection that affects the scalp, eyebrows, and eyelashes. It is caused by the Trichophyton and Microsporum fungal species, which attack the hair shafts and follicles. Antifungals are primarily used to treat this condition, with oral griseofulvin being the traditional drug of choice.

An 8-year-old boy presents to the office due to patches of hair loss to the scalp that began about 2 weeks ago and has gotten progressively worse. The patches are red and occasionally itchy. He does not report itching currently. On physical exam, there are several well-circumscribed pale, scaly patches with broken stubs of hair sticking up within each patch. Which of the following is the most likely diagnosis? AAlopecia areata BSeborrheic dermatitis CTinea capitis DTrichotillomania

DNasal polyps Dullness to percussion (C) is found in patients with consolidation (e.g., pneumonia), whereas patients with cystic fibrosis demonstrate hyperresonance to percussion.

An 8-year-old boy presents to the pediatrician's office due to a chronic, worsening cough with recurrent hemoptysis and decreased exercise tolerance. Laboratory results indicate an elevated sweat chloride level of 72 mmol/L. Which of the following is most likely found on physical exam? ACyanosis BDecreased anteroposterior chest diameter CDullness to percussion DNasal polyps

D Poor lung expansion with ground-glass appearance bilaterally Hyaline membrane disease, also known as respiratory distress syndrome, is a condition that may be found in premature infants.

An infant boy born at 32 weeks gestation is found to have tachypnea, expiratory grunting, intercostal retractions, and nasal flaring at birth. Nasal continuous positive airway pressure is implemented with immediate improvement in symptoms. Which of the following findings will most likely be found on obtaining a chest radiograph? ABilateral pulmonary interstitial opacities BIncreased bilateral pulmonary vascular markings and cardiomegaly CPneumothorax DPoor lung expansion with ground-glass appearance bilaterally

BHydrocortisone, fludrocortisone, and sodium chloride Most cases of congenital adrenal hyperplasia are caused by a mutation of the CYP21A2 gene. This is an autosomal recessive genetic disorder that causes deficiency of 21-hydroxylase necessary for the conversion of 17-hydroxyprogesterone to 11-deoxycortisol. deficiency in cortisol and aldosterone with an increase in adrenal androgens. Genital atypia, such as labial fusion, urogenital sinus, and clitoral enlargement, is common, and infants with genital atypia should be thoroughly examined for congenital adrenal hyperplasia. treatment should be initiated with hydrocortisone, fludrocortisone, and sodium chloride.

An infant is born with atypical genitalia characterized by clitoral enlargement, a common urethral-vaginal orifice, and labial fusion. Genetic testing is performed, and the infant is found to be 46,XX. Serum testing shows an increase in 17-hydroxyprogesterone. Which of the following is the most appropriate therapeutic regimen for the most likely diagnosis? ADexamethasone and fludrocortisone BHydrocortisone, fludrocortisone, and sodium chloride CPrednisolone and testosterone DPrednisone, hydrocortisone, and sodium chloride

birth fractures, bowing of the extremities, scoliosis, Wormian bones and basilar deformities of the skull, and short stature. Extraskeletal findings include dental abnormalities (dentinogenesis imperfecta), which affect the primary teeth more than the secondary teeth, easy bruising, hyperlaxity of the skin and ligaments, and conductive and sensorineural hearing loss. Blue sclerae

osteogenesis imperfecta PE

Healthy: 5-84th Overweight: 85-94th Obese: >/= 95th

BMI and weight Healthy: Overweight: Obese:

Neonates: Group B strep and E.coli older infants-children <5: viral ** >5: Strep pneumo TX: Amoxicillin is the drug of choice for empiric, outpatient treatment of children aged 6 months and older with suspected typical bacterial pneumonia (e.g., Streptococcus pneumoniae). Children older than 5 years with signs and symptoms of Mycoplasma pneumoniae or Chlamydia pneumoniae infection should alternatively be treated with a macrolide antibiotic (i.e., azithromycin, clarithromycin, erythromycin), doxycycline, or a respiratory fluoroquinolone (i.e., moxifloxacin, levofloxacin).

MCC of pneumonia in neonates, older infants-children <5 years of age, and in children >5 years of age and tx?

BDopamine and norepinephrine reuptake inhibition

MOA of the most commonly used medication for ADHD? AAlpha-2 receptor agonist BDopamine and norepinephrine reuptake inhibition CSelective serotonin reuptake inhibition DSerotonin and norepinephrine reuptake inhibition

Cervical venous hum

This murmur is heard best over the left or right upper sternal borders, infraclavicular area, or supraclavicular area. It is a continuous murmur that is accentuated with head extension while seated

anterior septum Kiesselbach plexus

Epistaxis, more commonly known as a nosebleed, is a commonly seen condition in children and may present due to trauma to the _________ in an area known as the ____________________

types 2 & 4

HPV strains that cause common warts

HPV type 1: plantar warts HPV 6 & 11: genital/anogenital HPV 16 & 18: cervical/anal cancers

HPV type 1 HPV 6 & 11 HPV 16 & 18 associations?

familial short stature and constitutional delay of growth and puberty. Pathologic causes include Turner syndrome, inflammatory bowel disease, malabsorption, malnutrition, and hormonal abnormalities.

What are the 2 most common nonpathological causes of short stature?

symmetric pattern on the palms, soles, backs of the hands and feet, and the extensor aspect of the forearms and legs.

How and where does erythema multiforme show up on the body

Erythema multiforme

What cutaneous reaction is characterized by targetoid lesions with a negative Nikolsky sign, often affecting the palms and soles?

Rubella

What in utero infection is associated with a patent ductus arteriosus?

bone conduction will be greater than air conduction in the affected ear. BC>AC (lateralization to affected side) Sensorineural AC>BC (lateralization to nl/better hearing side)

In conductive hearing loss, the sound will lateralize to the affected side. In sensorineural hearing loss, the sound will lateralize to the unaffected side. A normal result is air conduction greater than bone conduction. For conductive hearing loss which is greater? BC or AC for Sensorineural whic his greater?

Adenovirus

What is the MCC of viral conjunctivitis?

Ventricular septal defect.

What is the most common congenital heart defect?

Parvovirus B19 (fifth disease)

It begins with fever, headache, nausea, diarrhea, and coryza. Then, 2 to 5 days later, an erythematous malar rash with circumoral pallor presents, followed several days later by a reticulated or lace-like rash on the trunk and extremities.

AAvoid deliberately manipulating the lesion TOC: salicylic acid or cryo

What is the treatment of choice for patients with warts? AAvoid deliberately manipulating the lesion BReceive 9-valent human papillomavirus vaccination CReceive bleomycin injection

pastia lines-- scarlet fever

Linear petechiae over the axillary folds are also called

chronic plaque psoriasis

MC psoriasis in children?

Parvovirus B19 (erythema infectiosum) fifth disease

Most commonly seen in school-aged children and is associated with a slapped-cheek rash, an erythematous malar rash with circumoral pallor. The rash typically appears a few days after a prodrome of fever, headache, sore throat, cough, nausea, diarrhea, or conjunctivitis and is followed by a reticular rash on the trunk and extremities.

Increased tactile fremitus, positive egophony, dullness to percussion

PE signs of pneumonia

introduction of cows milk <1 year of age, insuff iron intake, obestity patient is presenting with IDA PE can also show atrophic glossitis or koilonchychia spoon nails

Patient comes in with increased total iron binding capacity TIBC, decreased hgb, (microcytic, hypochronic) sx: fatigue, SOB, tachy what are the mc dietary issues that contribute to pts condition?

intussusception DX: US TX: Barium contrast enema

Patient will be a child 6 months to 3 years old Colicky abdominal pain, vomiting, and bloody stools (currant jelly) Diagnosis is made by ultrasound (target sign). The patient commonly draws up their legs toward their abdomen and cries inconsolably. DX, TX?

Duodenal Atresia Plain abdominal radiographs also show a double bubble sign. Duodenal atresia must be corrected surgically by performing a duodenoduodenostomy.

Patient will be a newborn within the first day of life PE will show recurrent bilious vomiting without abdominal distention Plain radiograph will show double-bubble sign

BLorazepam

Status epilepticus (>/= 5 mins of cont seizure) first line treatment ALevetiracetam BLorazepam CPhenytoin DValproic acid

What is the organism that most commonly causes a peritonsillar abscess?

Streptococcus pyogenes

CT scan of the lungs, not gram stain culture bc Gram stain culture of sputum (C) will most likely show the Pseudomonas bacteria, as the patient has cystic fibrosis. Although helpful in the diagnosis of bronchiectasis, Gram stain and culture are not the best tests to confirm the diagnosis.

Test of choice to confirm dx of bronchiectasis in pt with a hx of CF presenting with brown foul sputum after cough?

with mild hearing loss between 20 and 40 dB lost, moderate with 40-60 dB lost, severe with 60-80 dB lost, and profound hearing loss as > 80 dB lost.

The American National Standards Institute defines hearing loss by the number of decibels (dB) lost, what is mild, moderate and severe hearing loss?

Penicillins

Which medications, when given to patients with mononucleosis, are known for causing a characteristic prolonged, pruritic maculopapular rash?

DValsalva maneuver (decreases preload) Physical examination may be normal or reveal a bisferiens carotid pulse, triple apical impulse, and loud fourth heart sound. Obstruction causes a loud crescendo-decrescendo systolic murmur heard best at the left sternal border. This murmur is accentuated with upright posture or Valsalva maneuver and decreases with squatting. Mitral regurgitation may also be noted on cardiac auscultation. Echocardiography is the test of choice to diagnose hypertrophic cardiomyopathy with left ventricular wall thickness > 1.5 cm. First-line symptomatic treatment includes beta-blockers (e.g., propranolol, metoprolol).

Which of the following will increase the intensity of the murmur associated with hypertrophic cardiomyopathy? AHandgrip BSquatting CTrendelenburg position DValsalva maneuver

CPalivizumab Palivizumab is a humanized monoclonal antibody against a respiratory syncytial glycoprotein that decreases the risk of hospitalization in children who were born prematurely, have a chronic lung disease, or have a significant congenital heart disease. It is given as a series of injections.

Which of the following would be most useful in preventing recurrence of RSV in a patient? AInhaled glucocorticoid BMontelukast CPalivizumab DRespiratory syncytial virus vaccination

60 mEq/L

an elevated quantitative sweat chloride test of >___ is definitive of CF

Croup (laryngotracheobronchitis) mcc parainfluenza virus steeple sign on xray tx: steroid, racemic epi

barky, seal like cough

Slate gray nevi, also known as congenital dermal melanocytosis.

benign hereditary congenital condition found in healthy infants and refers to the bluish-gray macules that develop on the sacrococcygeal area.

ACochlear implantation

best tx for sensorineural hearing loss in 2 day old infant that has hearing loss on neonatal hearing screening? ACochlear implantation BMyringotomy tube placement CStapedectomy DTympanoplasty

indirect which is mc type of hernia in kids

bulge in scrotum-- indirect or direct inguinal hernia

Wilson's disease

causes unconjugated hyperbilirubinemia due to increased bilirubin production. Patients commonly have a combination of hepatic, neurologic, and psychiatric symptoms, with onset as early as 5 years old. The hallmark diagnostic finding for this disease is Kayser-Fleischer rings on slit-lamp examination.

CReduced mobility of the tympanic membrane with pneumatic pressure On physical exam, the tympanic membrane will be erythematous and bulging, and when using a pneumatic otoscope, the tympanic membrane will have reduced mobility. The first-line treatment for children is amoxicillin. For patients with a penicillin allergy, a cephalosporin may be used, such as cefdinir or ceftriaxone, or a macrolide, such as azithromycin.

characteristic of acute otitis media on PE? ABright light reflexes to the tympanic membrane BErythematous ear canal with white debris CReduced mobility of the tympanic membrane with pneumatic pressure DTenderness to palpation over the mastoid bone

otits media

common complication of influenza

Conduct disorder is a disruptive behavior disorder in children characterized by a repetitive and persistent pattern of defiant, dissocial, and aggressive behaviors that violate age-appropriate social norms. Oppositional defiant disorder is similar to conduct disorder in that children are defiant, disobedient, and hostile toward authority figures, however, they do not violate societal norms or rules and are not violent toward other people or animals. 1/3 of children with oppositional defiant disorder progress to conduct disorder

conduct d/o vs ODD

viral croup responds well to nebulized epi BT does not in BT the patient will present more severe and toxic

difference between viral croup and bacterial tracheitis?

Ethosuximide

drug of choice for absence seizures

Rocky mountains spotted fever tx is DOXY always

erythematous macular rash on bilateral wrists and ankles, rash is blanchable. HA and fever x 3 days. What is dx and tx?

inhaled beta 2 agonist such as albuterol inhaled tiotropium LAMA for long term maintenance therapy subQ terbutaline option for acute exacerbations for those really ill and not first-line

first line treatment for acute asthma

Permethrin

first line treatment for lice or pediculosis capitis

spirometry

how is asthma diagnosed aka what test

D Hypokalemic, hypochloremic metabolic alkalosis The patient in the vignette has pyloric stenosis, which is a functional outlet obstruction that typically presents between 3 to 6 weeks of age. DX: US-- target sign

immediate, projectile, postprandial, and nonbilious vomiting. palpable nontender olive-shaped mass at the right upper quadrant of the abdomen. what would be seen in lab findings? AHyperkalemic, hyperchloremic metabolic acidosis BHyperkalemic, hypochloremic metabolic alkalosis CHypokalemic, hypochloremic metabolic acidosis DHypokalemic, hypochloremic metabolic alkalosis

Vertex - crown of the head highest point

in pts with alopecia areata where are the earliest locations of hair loss present on scalp?

The Centers for Disease Control and Prevention recommends universal annual vaccination for all patients ≥ 6 months of age. Children 6 months through 8 years of age who have not been previously vaccinated should receive two doses of influenza vaccine 4 weeks apart during the first season to optimize their immune response.

influenza recommendation

roseola

initially presents with a fever that may exceed 104°F and lasts for 3 to 5 days followed by a blanching macular or maculopapular rash beginning on the neck and trunk then spreading to the face and extremities. Occasionally, the rash is vesicular. It typically persists for 1 to 2 days, but it can come and go within 2 to 4 hours.

still murmur

innocent cardiac murmur with a vibratory or musical quality that is heard best over the lower left sternal border. This murmur commonly resolves spontaneously by early adolescence.

scarlet fever

is due to infection with streptococcus that presents with a rash that is diffusely erythematous, blanches with pressure, and is elevated, which gives a sandpaper-like quality to the skin. It usually starts in the groin or armpits before moving to the trunk and extremities. It is accompanied by circumoral pallor and a strawberry tongue, and it ultimately desquamates.

patent ductus arteriosus

machine-like and continuous. This conduit generally closes within 48 hours of birth in term infants.

meningitis**

macular non-blanchable rash r/o what??

CStaphylococcus aureus Pediatric patients with indwelling central venous catheters and patients with rheumatic heart disease are at increased risk. Signs and symptoms of acute infective endocarditis include rapidly deteriorating hemodynamic stability, tachypnea, tachycardia, altered mental status, new cardiac murmurs, and signs of systemic embolization, such as hematuria, pneumonia, osteomyelitis, and decreased perfusion to the extremities. Definitive diagnosis of infective endocarditis is made using the modified Duke criteria

mc organism causing infective endocarditis AEscherichia coli BHaemophilus influenzae CStaphylococcus aureus DStreptococcus pneumoniae

1-3 mos: group B strep and gram - bacilli 3-10 y/o: strep pneumo >/= 10 years: Neisseria meningiditis

mc pathogens meningitis 1-3 mos: 3-10 y/o: >/= 10 years:

Osgood-Schlatter disease

pain over the tibial tubercles bilaterally or osteochondrosis of the tibial tubercle

paraphimosis PE will show foreskin that, once retracted, cannot be brought to the usual position. Swollen glans penis with a constricting band just proximal to the corona phimosis swollen glans penis and foreskin that cannot be retracted

paraphimosis vs phimosis

4mlx TBSA burned x body weight kg (1/2 of this given in first 8 hrs with remaining given over the next 16 hours)

parkland forumula

CRepetitive strain at the tibial tubercle apophysis AKA osteochondrosis of the tibial tubercle. The patellar tendon inserts on the tibial tubercle, which is a secondary ossification center that begins to ossify around the time of the adolescent growth spurt, typically between the ages of 10 and 13 in girls and 12 to 15 in boys. Repeated contraction of the quadriceps muscles, such as in running, cutting, and jumping, puts tension on the patellar tendon. This repetitive tension produces strain at the tibial tubercle apophysis, resulting in chronic avulsion. The ossified portion of the apophysis pulls away from the tibial tubercle, and fibrous and fibrocartilaginous tissue fills the defect.

pathogenesis of osgood-schlatter disease AAvulsion of the anterior tibial spine BInflammation of the patellar tendon CRepetitive strain at the tibial tubercle apophysis DStress fracture of the tibia

ASD

presents with a systolic murmur heard best at the left upper sternal border. It is usually grade 1 or 2 in intensity and results from increased flow across the pulmonary valve secondary to right ventricular volume overload. A wide, fixed, split second heart sound may be the only finding associated with this congenital heart defect.

DOXY pt is presenting with Lyme arthritis (MC presentation of late lyme dz)

pt presents with swelling and stiffness in knee, afebrile, able to bear weight, moderate pain, joint effusion present. Synovial fluid WBC count is elevated in the 20,000 to 60,000 range (not as high as septic arthritis). what is the tx for pts illness?

Crigler-Najjar syndrome (A)

rare disorder of bilirubin metabolism in which infants develop persistent jaundice within the first few days after birth, which can lead to kernicterus. Bilirubin levels commonly range from 8-20 mg/dL but can rise to 50 mg/dL during illness or fasting in later life.

Osteosarcoma Any mass > 4 cm in diameter, or the size of a golf ball, should raise suspicion for malignancy. X-rays show destruction of bone, periosteal reaction, poorly defined margins, and a soft tissue mass that may be calcified.

rare malignant bone-forming tumors but are the most common primary bone tumor in children and young adults. starburst pattern on xray

PDA prematurity, female, & being born at high altitude

risk factors for machine like continuous murmur

lethargy, coaurse facies, hoarse cry, hypotonia, dry skin, hypothermia

s/sx of congenital hypothyroidism

- obs - bracing - surgery - diagnosis of scoliosis is made by Cobb angle >/= 10 degrees

scoliosis angles and treatment - between 20 and 40: - between 25-39: - >/= 50:

Nephrotic syndrome

shows many symptoms similar to poststreptococcal glomerulonephritis, as an aspect of the latter includes the former. While proteinuria is seen in the child in the vignette, due to the previous infection, the correct answer is poststreptococcal glomerulonephritis. Urinalysis with this condition would likely include oval fat bodies and not red blood cell casts.

immediate ER transfer... Epiglottitis is inflammation of the epiglottis and adjacent supraglottic soft tissue structures, most often secondary to infection with bacterial pathogens from the posterior nasopharynx. HIB***

sore throat, stridor, leaning forward, drooling fever, sore throat...

**MC cyanotic cardiac dz** P-pulmonary stenosis R-RVH O-overriding aorta V-VSD E- echo dx (boot shaped heart on xray) harsh systolic crescendo-decrescendo murmur at the left sternal border, a palpable thrill, and a prominent right ventricular impulse.

tetralogy of fallot (PROVE)

Oral amoxicillin or augmentin x 10 days

tx for TM perf

Reasurrance, resolves spontaneously by 1 year of age. pharmaco theraoy not recommended for most infants w GERD.

tx for babies regurgitation <1 years of age and not failing to thrive.

rigid bronchoscopy

tx for foreign body aspiration

Mild: steroids, racemic epi Severe: transfer to ER

tx for laryngotracheitis (croup) mild and severe

BCiprofloxacin-hydrocortisone otic drops alone The most common pathogen responsible for otitis externa is Pseudomonas aeruginosa. Patients with otitis externa commonly present with ear pain, discharge from the ear canal, and itchiness to the ear. Treatment for moderate otitis externa is with a topical combination of an antibiotic and glucocorticoid, such as ciprofloxacin-hydrocortisone or neomycin-polymyxin B-hydrocortisone. Severe disease consists of cases with complete occlusion of the ear canal secondary to edema and intense pain and pruritus. Severe otitis externa is treated with the same topical drops as moderate disease, with an addition of a wick placement. If there is deep tissue involvement, then systemic antibiotics, such as ciprofloxacin or ofloxacin, should also be considered.

tx for mild otitis externa with partial visualization of tm clear with good light reflexes AAcetic acid-hydrocortisone otic drops alone BCiprofloxacin-hydrocortisone otic drops alone CCiprofloxacin-hydrocortisone otic drops and oral ciprofloxacin DNeomycin-polymyxin B-hydrocortisone otic drops and a wick placement

IV abx + myringotomy **if complicated then IV abx + mastoidectomy

tx for uncomplicated mastoiditis

Acute tubular necrosis

type of acute kidney injury often due to kidney ischemia, drug toxicity, or endogenous diseases. Urinalysis will show epithelial casts or muddy brown casts but not red blood cell casts, as seen in glomerulonephritis. Also not associated with prior Streptococcus infection.

viral will most likely have fever and preauricular lymphadenopathy allergic more likely to have itchiness and stringy discharge

viral conjunctivitis vs allergic conjunctivitis

Treatment for children > 12 months of age begins with vitamin D 50 mcg daily for at least 6 weeks followed by maintenance dosing of 600 to 1,000 international units (15-25 mcg) once daily.

vitamin D deficiency dosing

CRASH and burn CRASH and burn: conjunctivitis, rash, adenopathy (cervical lymphadenopathy), strawberry tongue, hand or foot edema, uncontrolled high fever >/= 5 days

what 4 out of 5 things + fever is needed to dx kawasaki dz

Bleeding when scraping off plaques

what happens when you try to scrape off plaques in a pt with oral candidiasis?

TEN is >30% while stevens-johnson is <10% of total body surface area involved & TEN includes mucosal lesions.

what is the difference between TEN and stevens-johnson syndrome?

cryotherapy

what is the first line tx for this rash?

parainfluenza virus and X ray finding is the steeple sign due to narrowing of the trachea.

what organism causes croup?

CGeneralized 3 Hz spike-and-wave discharges

what will an eeg show for absence seizures? ABilateral independent period discharges BBilateral synchronic and symmetric slow activity CGeneralized 3 Hz spike-and-wave discharges DLateralized periodic discharges

by 4 months

when do infants double their birth weight?

Whirlpool sign of mesentery can be seen on ultrasound in patients with malrotation or intestinal volvulus. The whirlpool sign is caused by the mesenteric blood vessels twisting around the base of the mesenteric pedicle.

when is whirlpool sign seen?

extreme pain/TTP and absent cremasteric reflex

which PE finding presenting with testicular torsion?

Enterobius vermicularis

which parasites cause pinworm infection


Related study sets

Unit 29: Three - Phase Transformers

View Set

Common Classroom Questions & Phrases (Español I)

View Set

Leccion 14.1 and 14.2 SPA 201 Study Exam

View Set

Harr MLS Review Chemistry 5.4 Calculations, Quality Control, and Statistics

View Set

Psych exam 2 Chapter 6 Practice questions

View Set